Which of the following findings would most weaken the authors explanation of the observed effects of class size reductions?

Exam 3 Psychology/Sociology Section Passage 1

1) The P/S section on the MCAT doesn’t get much love. Students often brush it to the side and assume they can brute force their way through it. One of the most common statements I’ve heard students make about this section has been “Oh, I just need to sit down and memorize more information.” While this is more true than other sections, critical thinking and reasoning are still very much at play in this section, and like we’ll see, this question.

The question is asking us to identify a limitation specific to a 5 year old that will negatively impact the child’s ability to do well in the studies regarding perspective-taking and empathy. What would prevent a child from being empathic and taking on others’ perspectives?

  1. Children lack object permanence up until about 2 years of age, or the end of the sensorimotor stage, according to Piaget’s stages of development. Note that while the question made no mention of a specific developmental theory, it did specify a cognitive limitation which points towards Piaget’s theory of cognitive development. Even if we were unsure at what age children gain object permanence, a lack of object permanence is not directly associated with the ability to care about others and their perspectives.
    For more on cognition, visit the corresponding content page: https://jackwestin.com/resources/mcat-content/cognition/cognitive-development
  2. According to Piaget’s theory of cognitive development, conservation develops during the concrete operational stage which spans from about 7 to 11 years of age. We would expect a 5 year old to have limited understanding of conservation. If we only used our content knowledge, we would be tempted to select this answer choice on test day and move on. However, a strong understanding of conservation does not impact someone’s ability to empathize. Recall that conservation means recognizing that the quantity of something remains the same even if the way it’s presented changes. The classic example of this is recognizing that the amount of water in a glass does not change if you pour it into a shorter and wider container.
  3. Centration is a cognitive limitation present during the preoperational stage of development which spans 2 to 7 years of age, meaning it applies to a 5 year old. Centration corresponds to a fixation, or centeredness, on a single component or aspect of a larger whole. An example of this would be a child that sees that your shirt has a blue button instead of noting that the button is a part or a larger shirt that is long sleeved, has alternating colored buttons and a checkered floral pattern. The child has focused on a single detail and is unable to see the larger whole. This answer choice is getting closer to what we’re looking for, but this centeredness is a more detail- or sensory-oriented centeredness rather than one based on the ability to take someone else’s perspective. There is a better answer choice.
  4. This is the correct answer. Like centration, egocentrism is a marker of the preoperational stage of Piaget’s theory of cognitive development and would apply to a 5 year old playing the researchers’ games. Egocentrism is a focus on oneself rather than others and the inability to take on the perspective of others. This would directly impact a child’s ability to empathise (affective) and take on someone else’s perspective (cognitive), thereby limiting their performance on these dependent variables.

2)

  1. People are attracted to others who enjoy similar activities, making this answer choice incorrect for a LEAST question. You’d be hard pressed to find someone with a close friend that doesn’t like any of the same activities; they wouldn’t have anything to do together!
  2. Like answer choice A, people tend to be attracted to people they find physically attractive.
  3. Familiarity can encourage fondness and interpersonal attraction. Think back to grade school, most of us chose children that we knew or had worked with before when teachers told us to find a partner. Running into someone before the study and then seeing them again during the study would make a participant more likely to choose them which would influence interpersonal attraction and make this answer choice incorrect.
  4. Neuroticism is one of the personality traits likely to come up in discussions of the “Big Five” and “PEN” trait theories; it is the propensity to become anxious or insecure in stressful situations. Neuroticism will not predict interpersonal attraction making it the correct answer. This is where many students might be tempted to justify this answer by saying that being anxious and insecure might affect someone’s decision making abilities. It is true that anxiety can and often does impact decision-making, however if you’re deciding who to team up with between two people, neuroticism and that propensity towards anxiety will not make you more likely to choose one person over the other.

3)

  1. According to the hypothesis in the first paragraph, empathy and perspective-taking are two related but separate skills. This suggests that someone who is empathic should also be good at perspective-taking and their scores on the two measures should track in the same direction which would correspond to a positive correlation. When the results are as expected, in this case there is indeed a positive correlation between two measures known to be related, then there is no immediate concern regarding the validity of the measures.
  2. If someone scores highly empathic but low on distress when watching others in pain or vice versa, the researchers should be scratching their heads and should question the validity of the measures. Seeing someone else in pain or distress will produce some corresponding distress in someone who is empathic. A lack of congruence between the two measures of empathy should be a red flag. These two measures should track in the same direction; when they do not, the validity of both measures is likely compromised making this the best answer choice.
  3. According to the passage, empathy is affective and perspective-taking is cognitive. That said, the fantasy component of the perspective-taking questionnaire specifically asks about getting involved in the feelings of a character. A positive correlation between empathy and the cognitive-emotional component of perspective taking would be expected; validity is not in doubt here.
  4. Recall that study 1 focused on perspective-taking and study 2 focused on empathy. We are told in the first paragraph that the hypothesis is that perspective-taking and empathy are related but separate. If no correlation were found between the empathy questionnaire scores and the amount of money won during the competition trials of the perspective-taking study, we would not have reason to question the validity of the measures. The competition trials rewarded participants money regardless of the opponent’s action so the participants did not need to consider the perspective of the other person; the competition trials did not measure perspective-taking. While empathy and perspective-taking are related, the competition trials do not fall under the umbrella and a correlation is not expected so the lack of correlation does not point to a lack of validity. Answer choice B provides a direct reason to question the validity of the measures making it the best answer choice.

4)

  1. The cerebral cortex is what most people think of when they hear “brain.” The cerebral cortex controls our consciousness, thinking, planning, integration of sensory information, memory, motor function and more just to name a few. A thickening of the cortex, even if it did correlate with improved performance on both studies, would not actually address the researchers’ hypothesis that empathy and perspective-taking are two related but separate processes. This answer choice lumps the two studies together and relates the entire cortex to both studies. In order to support the hypothesis that empathy and perspective-taking are related but separate, there should be something in the hypothetical findings that differentiates the two skills.
  2. This answer choice is tempting for many students because our minds often fill-in-the-blanks to encourage our engagement with the material in front of us. Metaphorically, raise your hand if you imagined the participants having face-to-face interactions. Now, does it actually say this anywhere in the passage? Surprisingly, it does not. It is reasonable to assume that the participants in study 2 had some face-to-face interaction in order to get to know each other and so it’s plausible that difficulty recognizing faces might impair their ability to empathize with someone else and form a coalition, but this is still speculation. Even more importantly, there is no mention of participants seeing each other’s faces in study 1, and this study could easily be performed without the participants seeing one another (this happens frequently in video games, or, and I’m aging myself here, a round of Battleship with your face hidden behind a divider so that your opponent can’t read your facial expressions). If the participants weren’t looking at one another to begin with, then difficulty with facial recognition should not impact their performance. I want to pause and recognize that we engaged in a lot of hypotheticals with this answer choice, and that’s precisely part of the point; the more hypotheticals and assumptions you have to make to select an answer, the less likely it is to be correct. For this answer choice to be correct, we would have to make at least three leaps of logic: 1) study 1 involves the participants looking at one another, 2) study 2 also involves the participants looking at one another and 3) recognizing someone else’s face is related to empathy but not perspective-taking. There has to be a better answer choice more grounded in the passage.
  3. If this were true, there would be a problem with the design of the studies. If the participants of a study cannot understand the instructions of that study, the results cannot be taken at face value and the study has poor validity. Additionally, there is no reason to believe that perspective-taking (study 1) has an auditory component not found in empathy (study 2).
  4. This answer choice is the best answer because it supports the idea that empathy and perspective-taking are separate. If a specific region of the cortex is damaged and only one of the two skills is affected (empathy because the participants have difficulty with study 2), then the damaged region is likely related to the impaired skill and not involved in the unimpaired skill (perspective-taking in study 1).

Exam 3 Psychology/Sociology Section Passage 2

5)

  1. Reproductive memory is a type of idealized memory where we recall information exactly the way it occurred, or reproduce it with high fidelity. This is very unusual because our recall memory of events is often flawed.
  2. Flashbulb memories are memories that are especially vivid, memorable and more likely to be remembered with greater detail due to a strong emotional associations. Particularly bad news, major life events and societal turning points are classic examples of flashbulb memories. The famous political assassination attempts in study 1 and the disasters in study 2 are examples of flashbulb memories.
  3. As the name implies, prospective memory is a memory oriented towards the future, such as the memory of planning a future action or the memory of an intention for the future. Many of you have prospective memories planning your future careers. The memories in study 1 and study 2 were not regarding planning the future and are not prospective memories.
  4. Eidetic memory is colloquially known as photographic memory and is the ability to recall an image with near perfect or high accuracy after seeing the image once and for a short period of time. This does not apply to the vivid memory of emotionally-associated events as the ones in studies 1 and 2.

6) The second paragraph simply mentions that the participants were asked about key details regarding how they heard about a disaster. If you decide you need to, ahem, refresh your memory after going through the following explanations, check out https://jackwestin.com/resources/mcat-content/memory/storage

  1. Episodic memories are related to events and experiences; knowing the details of the event and remembering how one experiences the event and the experience of learning about the event are examples of episodic memory making this the best answer. These memories generally have details that help provide context such as timing, location and other associated experiences. A concrete example would be remembering the time you broke your foot while riding your bike. You were 7 years old, biking outside your home during the afternoon following school and lost control trying to go over a curve. You then went to the hospital and got a cast. Here the “tags” or specific details that you might share with someone when asked about the incident are how old you were, the place, time of day and subsequent events.
  2. Semantic memories are common knowledge or concepts. While the disasters might eventually become a part of the common knowledge, details about how the individual participant found out about the disaster and even details of the event itself are not a part of common knowledge. Better examples of semantic memories are that the sky is blue and that 4+4=8.
  3. Procedural memories are memories that correspond to skills and tasks such as riding a bike, not the specific details of an event.
  4. Iconic memory, like the name implies, relates to an icon or an image. While the participants might have had a specific image that they associated with the event, they were asked about details of the event and their memory of learning about the event, not any images they might have associated with the event. This is best described as episodic memory, not iconic memory.

7)

  1. It would be more probable that false information was encoded if that same information had been provided during the initial description of the memory of the event rather than during the second recollection. Something about sharing the details of the event the second time accounts for the intrusions.
  2. Prospective memory refers to memories of planning and future events. The participants are asked to share their memories of past events, not plans for future events. This means that the memory relevant to the study and therefore the question is not prospective.
  3. In the last few sentences of the third paragraph, the author notes that the intrusions corresponded to getting the key details “wrong,” or to providing false information even though the information was consistent with the situation or type of event. This suggests that retrieving the memory for a second time did not only retrieve the stored episodic memory, but also retrieved memory of general knowledge and of similar events. This is a consequence of the reconstruction of memories and is part of the reason why it can be so difficult to accurately reconstruct memories. This is the correct answer.
  4. This answer choice is unlikely because the intrusion errors were just that, errors. They were incorrect key details even though they aligned with the general situation. If they had been encoded during the initial event but repressed, they could appear later but would still be incorrect. However, the participants had initially described the correct key details, suggesting that they encoded the correct details as a part of their episodic memory, not the incorrect details as would have to be the case for repressed, incorrect information to be retrieved the second time.

8) Recall that generalizability refers to the ability to extrapolate and apply the results of a study or research to “real life” or other places, people and scenarios.

  1. This would affect the internal validity, or the ability to draw conclusions from the data, more than the external validity or the generalizability of the study. It is possible to have laboratory results that mimic what occurs in the real world without knowing which of the specific variables tested account for the bulk of the results obtained. While not ideal for a laboratory study, this answer choice alone does not immediately jeopardize generalizability.
  2. Factual information addressed in laboratory studies could still be generalizable to factual information in the “real world.” The inclusion of emotions is not required as the two can be intertwined but they can also be distinct.
  3. If this were true, it would correspond to better generalizability because the measurement is more accurate and therefore more likely to be applicable to or representative of what happens in settings outside of the laboratory.
  4. This is the correct answer; it would cause concern for the generalizability of laboratory studies. If “real life” results are very different from the ones generated in laboratory settings, then the results obtained in the study are unlikely to match the ones produced outside of the laboratory making the results less generalizable.

9)

  1. This is the correct answer. Per the last sentence of the passage, a memory associated with strong emotions, such as a car crash, causes increased memory of central details and decreased memory of peripheral details. This is because the central details that actually trigger the emotional response are preferentially noticed/the focus of our attention. This helps us encode and remember the “most important” details. For example, if you’re attacked by a bear in the woods, it’s more important to notice and remember the presence of the bear and less important to notice and remember what color moss was on the rock behind the bear.
  2. The last paragraph does not mention an improvement in overall memory, but rather the shifting of improved memory for central details and less recollection of peripheral information.
  3. Peripheral details are still encoded; the passage notes fewer peripheral details remembered, not no peripheral details remembered at all. This suggests that the encoding of peripheral details is still intact and that the decrease in encoded peripheral information is likely the result of selective attention rather than an impairment in the encoding.
  4. The last paragraph does not mention anything to suggest that there is a difference in the encoding and the subsequent retrieval of specific types of details. Since there is no evidence of this in the passage, we’ll stick with answer choice A as the best answer to explain the finding from study 3 as noted in the passage.

Exam 3 Psychology/Sociology Section Discretes

10)

  1. Cones are photoreceptors responsible for the detection of bright light and daytime color vision. Individuals with impaired color perception are likely to have improper functioning of these weakly photosensitive receptors which detect the initial color stimulus. For more on the structures of the eye and their functions, check out https://jackwestin.com/resources/mcat-content/vision/structure-and-function-of-the-eye
  2. The cornea has two main functions: to serve as the lens for the eye and to protect the inside of the eye. While the lens function is responsible for the focusing of light rays and is involved in the formation of images onto the retina, this function does not include the detection of colors or color vision.
  3. Rods are the peripheral and nighttime vision counterpart to the cones. They are strongly photosensitive and detect dim light rather than colors. Improper functioning of the dim light detecting rods is unlikely to cause problems with color vision.
  4. The sclera of the eye is a source of protection against the outside world and helps maintain the shape of the eye, thereby complementing the role of the cornea. Neither of these two functions directly impact perception of color and as such are unlikely candidates for sources of impaired color vision. Answer choice A remains the most likely impaired component of the visual system in people with impaired color perception.

11)

Which of the following findings would most weaken the authors explanation of the observed effects of class size reductions?

Note: while results may be statistically significant even if the error bars overlap, for the purposes of the MCAT, assume that overlapping error bars or confidence intervals mean no statistically significant difference unless indicated otherwise.

  1. As shown above, this is incorrect because the monkeys fed a high-quality protein diet did have a feeding behavior preference even if the monkeys fed a low-quality protein did not.
  2. Assuming that protein insufficiency is the same as low-quality protein diet and that protein sufficiency is the same as the high-quality protein diet, this answer choice is also incorrect. As shown in the image above, low-quality protein did not influence future feeding behavior more than the high-quality diet. In fact, the monkeys fed a low-quality protein diet showed no significant future preference.
  3. This is correct; the monkeys fed a high-quality protein diet showed a significant future feeding preference indicating influence, while the monkeys initially fed a low-quality protein diet did not show a significant preference or influence.
  4. This is incorrect; the lack of significant difference in feeding behavior by the monkeys initially fed a low-quality protein diet shows that prior diet type is not solely responsible for feeding behavior. Also, it would be inappropriate to conclude that no other factors are determining feeding preference given the lack of information about the study design.

12) The 45 year-old man in the question stem arrived at the hospital with an acute presentation of symptoms that appeared to mimic a heart attack. Let’s look for an answer choice that would support a psychiatric explanation for symptoms mimicking a heart attack.

  1. Generalized anxiety disorder is, as the name suggests, quite general and encompasses a number of other disorders including but not limited to panic disorder, obsessive-compulsive disorder and post-traumatic stress disorder. Let’s see if we can find a more specific psychological disorder to explain the patient’s specific acute onset of symptoms.
  2. Panic disorder is characterized by panic attacks which can mimic heart attacks and may involve a sudden terror or intense fear, a quickened heart rate (tachycardia), sweating and chest pain. These symptoms, as well as their often acute presentation during a panic attack, are suggestive of a panic disorder.
  3. Illness anxiety disorder is the excessive concern and focus on symptoms or physical sensations and the severe worry that the person might be or will become sick. The patient in the question stem had an acute physical presentation that required the ruling out of a heart attack, but there was no mention of an excessive fear of illness.
  4. Somatic symptom disorder is characterized by the presence of physical symptoms that suggest a physical illness or the presence of physical injury but that is without a medical explanation. Because by definition the symptoms cannot be explained by medical testing, workup or medical diagnoses, the diagnosis of somatic symptom disorder is a diagnosis of exclusion. While the medical tests showed that the patient in the question stem did not have a heart attack, there is no mention of additional workup or other testing making somatic symptom disorder less likely to be the correct answer. Additionally, the set of symptoms experienced by the patient is a classic presentation of panic attacks meaning that the symptoms have an alternate medical explanation, ruling out somatic symptom disorder.

Medical school bonus tip: when you start learning about different possible diagnoses in medical school, you’ll learn to create what’s called a differential diagnosis which includes a list of possible medical explanations for a patient’s clinical presentation. You’ll then go through each possible diagnosis and explain why it is more or less likely given the patient’s presentation/signs and symptoms, and what you know about the conditions you listed. Well, you just started doing that with this question! Take a moment to enjoy working through this differential diagnosis and let’s keep tackling the remaining questions together!

A brief overview of some of the psychological disorders you might encounter on test day can be found at https://jackwestin.com/resources/mcat-content/psychological-disorders/types-of-psychological-disorders

13) The person in the question stem is awoken every time they show rapid eye movements which are specific to REM sleep. This means they are not getting REM sleep and therefore deep sleep.

  1. This is incorrect; sleep deprivation and interrupted sleep make it easier to fall asleep in the future. Many of us have experienced this: you get barely any sleep one night, are exhausted during the day and as soon as you get home, bam! Head on the pillow and lights out. The longer you go without sleep, the easier it becomes to fall asleep the next time.
  2. This is correct; REM sleep is particularly important and when REM sleep is interrupted and shortened, the body tries to make up for the lost REM sleep by subsequently increasing the frequency and duration of REM sleep in the future. This increase in frequency and duration are known as REM rebound. Take a peep at the states of consciousness on the content page: https://jackwestin.com/resources/mcat-content/consciousness/states-of-consciousness
  3. This is true of the night of the sleep study when the experimenter kept waking up the participant when they entered REM sleep. However, the following night, the participant should have more REM sleep than usual to make up for the REM deficit, not have even less REM sleep which would worsen the REM deficit.
  4. The body responds to sleep deprivation by increasing the frequency and depth of sleep moving forward, particularly making up for the REM sleep deficit by promoting REM rebound. The sleep pattern following the night of the sleep study should be different from the participant’s usual sleep pattern as a result of the REM rebound.

Exam 3 Psychology/Sociology Section Passage 3

14)

Which of the following findings would most weaken the authors explanation of the observed effects of class size reductions?

  1. This is correct. There is no asterisk and therefore no significant difference between any of the three therapies 6 months after treatment. All three therapies are equally effective at the 6 month follow-up.
  2. While CT does have a significantly higher value than CBT at the end of therapy, this means that the mean insomnia severity remains higher at the end of therapy, so it is less effective and not more effective.
  3. This is incorrect; at the end of therapy, CBT has a lower mean insomnia severity than CT indicating that CBT is more effective than just CT.
  4. All three therapies brought the serenity of insomnia below threshold levels at both the end of therapy and at the 6 month follow-up indicating that all three therapies were effective at both time points. Only answer choice A is an appropriate conclusion from the data.

15) According to the figure, the diagnostic criteria is noted by a threshold severity of 14 required for diagnosis.

Which of the following findings would most weaken the authors explanation of the observed effects of class size reductions?

  1. This statement is supported and therefore incorrect for this NOT question. At baseline, participants in all three therapy groups had similar severities hovering between 18 and 19 mean insomnia severity.
  2. This statement is true and therefore an incorrect answer for this NOT question. The threshold for diagnosis is noted as a mean insomnia severity of 14; all three therapies scored between 7 and 10 on the mean severity scale at the end of therapy which is below the diagnostic threshold of 14.
  3. This answer choice is also true and therefore incorrect. Like answer choice B, all three therapies were below the mean insomnia severity diagnostic score of 14 at the 6 month follow-up.
  4. Because this answer choice is false, it is the correct answer for this NOT question. The only data provided in the figure are for the therapies as groups; we are not given the information for any of the individuals within the therapeutic groups and as such cannot say that every individual within the group scored a specific value following therapy. Recall that means are averages of multiple values and two extremes (for example one very strongly responding participant and one unresponsive participant) can have the same average as two moderately responding participants.

16) If the researchers chose to use individual instead of group therapy to avoid the influence of something, then that influence must be related to or dependent on the presence of another individual (not the researcher but another participant).

A. Classical conditioning does not involve the presence of another participant meaning it is not the right answer. Classical conditioning involves the pairing of an unconditioned stimulus with a neutral stimulus to generate a conditioned stimulus that results in a predictable conditioned response.

Which of the following findings would most weaken the authors explanation of the observed effects of class size reductions?

https://jackwestin.com/resources/mcat-content/associative-learning/classical-conditioning

B. Observational learning requires the presence of another person making it a promising answer and in fact, the correct answer. Observational learning is very much as the name implies: the learning or acquisition of a behavior by watching or observing others model a behavior. This type of learning in group therapy could influence the results via a mechanism other than the type of therapy used.
https://jackwestin.com/resources/mcat-content/observational-learning

C. Operant conditioning does not require the presence of another individual receiving treatment to occur so this answer is incorrect. Operant conditioning uses punishments and rewards to increase the frequency of desired behaviors and decrease the frequency of undesired behaviors. Below you will find an example of operant conditioning.

Which of the following findings would most weaken the authors explanation of the observed effects of class size reductions?

https://jackwestin.com/resources/mcat-content/associative-learning/operant-conditioning

D. The elaboration likelihood model is concerned with how a message is delivered. The merits of the message itself constitute the central route of processing and while processing these components require more effort, they produce lasting change with respect to persuasion. On the other hand, cues that are associated with the message but not a part of the message, such as attractiveness or notoriety of the person delivering the message, require much less effort to process but are also more likely to result in a temporary attitude change. This model does not require the presence of a second study participant so answer choice B remains the best answer.

Which of the following findings would most weaken the authors explanation of the observed effects of class size reductions?

17) The cycle alluded to in the question stem is the cycle of worrying about sleepiness, that worry causing sleepiness, and then worrying even more about sleepiness which will continue worsening sleepiness… The concern is causing the object of concern to occur which worsens the initial concern.

  1. A self-fulfilling prophecy refers to the cycle where expectations, especially negative ones, influence the outcome and ultimately lead to the original expectation. A very common manifestation of self-fulfilling prophecies among students is being worried that they’re going to do poorly on an exam, their negative expectation and worry negatively impacting their ability to concentrate and that contributing to their actually doing poorly on the exam. This is very similar to the cycle described in the passage of worrying about sleep which causes having trouble with sleep which then worsens the initial worry. This is the correct answer. Content page: https://jackwestin.com/resources/mcat-content/processes-related-to-stereotypes-mcat-wiki/self-fulfilling-prophecy
    Note: it’s normal to be nervous about exams, especially for an exam like the MCAT. That said, it’s important to address and break cycles of negative thinking. Always keep in mind that you are learning, growing and that the MCAT is an exam for which you can prepare.
  2. Stereotype threat is the stress or anxiety produced when someone perceives themselves to be in a situation where they might confirm a negative stereotype about a group they belong to. The patients in the passage are not concerned with confirming any negative stereotypes so this answer choice is incorrect. For the associated content page: https://jackwestin.com/resources/mcat-content/processes-related-to-stereotypes-mcat-wiki/stereotype-threat
  3. The fundamental attribution error is quite common and occurs when someone attributes another person’s behavior to a disposition or internal factors rather than external or situational factors. For example, if someone cuts you off in traffic, you are more likely to say that they are rude and a bad driver than you are to say that they must be running late to an important meeting. The cycle of worry and sleepiness mentioned in the passage does not address whether someone’s action is the result of their character versus the context of the situation so this answer is incorrect. I would like to pause here and note that the fundamental attribution error is not the same as self-serving bias. While the fundamental attribution error emphasizes our assessment of someone else’s actions, the self-serving bias describes our assessment of our own actions such that if something negative occurs, it “wasn’t my fault” (external attribution) but if something positive occurs “that was all me” (internal attribution). For more on these and other attributional processes, visit: https://jackwestin.com/resources/mcat-content/attributing-behavior-to-persons-or-situations/attributional-processes
  4. Cognitive dissonance occurs when someone experiences tension between beliefs, attitudes or behaviors they hold, and suggests that the person will try to minimize that tension by either questioning the beliefs, rationalizing or denying the tension, or changing the behavior. Of note, changing the behavior is less likely to occur than questioning information or adding new pieces of information. A common example is a doctor who smokes; they believe their smoking can negatively impact their health but they smoke nonetheless. The cycle of worry and sleepiness mentioned in the passage is not the result of a contradiction between two or more ideas or behaviors and is therefore not cognitive dissonance. You can walk through an example of cognitive dissonance at https://jackwestin.com/resources/mcat-content/theories-of-attitude-and-behavior-change/factors-that-affect-attitude-change

Exam 3 Psychology/Sociology Section Passage 4

18) The question is asking us to identify a factor that is not a component of SES, or socioeconomic status.

  1. Occupational status and employment are key components of SES, they actively impact social status and economic circumstances.
  2. Like occupational status, source of income directly impacts a person’s economic status meaning it is a component of SES and incorrect for this LEAST question.
  3. This is the correct answer because social capital is not one of the primary components of SES. Social capital is the set of benefits provided as a result of social networks, or “the benefits of who you know” and the support systems derived from the people you know such as friends, community, etc. This is not a direct component of SES making it the least likely to be included as a measure of SES.
  4. Educational attainment can affect a person’s finances and their social standing meaning it would be included in a measure of SES and that this answer is incorrect. If you are still a bit unsure about SES, read through https://jackwestin.com/resources/mcat-content/social-class/aspects-of-social-stratification

19) According to the passage, individuals with a lower SES are disproportionately exposed to stress. This means that as SES decreases, stress increases and that the two measures are negatively correlated.

  1. This is the correct answer because it shows a negative correlation (lower SES corresponding to higher stress exposure) but is also large enough to be meaningful. On test day we would scan the rest of the answers in case there were other negative correlations and then assess the magnitude of those correlations as appropriate.
  2. This answer choice represents a negative correlation, but it is an incredibly weak association! Recall that an estimate of association of 0 means no association whatsoever; 0.05 is too close to 0 to represent a “disproportionate” exposure.
  3. This answer choice represents a positive correlation which would mean that stress increases as SES increases, but the passage notes that lower and not higher SES is associated with greater exposure to stress.
  4. Like answer choice C, this answer choice incorrectly captures a positive correlation whereas the relationship between SES and stress described in the passage is a negative correlation. Only answer choice A correctly captures a negative correlation of sufficient magnitude to describe a “disproportionate” exposure.

20) Looking at this question, you might be tempted to start digging around in the passage for mention of heart disease or start racking your brain for the statistics on depression you may have read when reviewing the psychological disorders for the P/S section. The reality is that the answer is like objects in a car mirror, “closer than they appear.” We will anchor ourselves in the last sentence of the passage that notes that the disproportionate exposure of people in lower socioeconomic positions to stressors can explain differences in mental and physical health. Combining that information from the passage with the question stem specifying anger and hostility, let’s find an answer choice that is a logical mental or physical health disparity that could result from stressors associated with anger and hostility.

  1. Heart disease would be a physical health disparity that could reasonably be associated with negative stressors, anger and hostility. Imagine this: you’re in the emergency room with a patient clutching their chest and having symptoms of a heart attack. They start screaming and running around the room, absolutely furious. Would you let the patient who appears to have heart problems keep running around in a fit of fury? No, you’d probably say something along the lines of “let’s get you into an exam room and help figure out what’s wrong. It’s not good for your heart for you to be running around all worked up.” Sometimes the best answers in the psych/soc section of the MCAT are the ones that you can see play out in your head. We would keep reading on test day but know that this is the correct answer.
  2. Stronger reliance on coping mechanisms could either be good or bad depending on if the coping mechanism is a healthy coping mechanism or not. This type of ambiguity makes for a poor answer choice. Additionally, the last sentence of the passage notes the presence of weak stress mediators, suggesting that even if the coping mechanisms are healthy coping mechanisms/stress mediators they are still weak. Someone who is actively angry and hostile is less likely to default to a healthy coping strategy, especially if it’s a weak strategy. Example time: if someone throws a trash can at you in the middle of a heated argument, are you more likely to respond by crying, yelling and/or storming out of the room, or are you more likely to softly ask them to sit while you go find a mediator? Most people would fall into the first category. We would have to make a lot of assumptions to make this answer choice work and honestly, even then it doesn’t quite make sense in most cases. Next!
  3. If there is a reduced external locus of control, then it follows that there is an increased internal locus of control. Imagining once again that you are angry because someone threw a trash can at you during an argument; do you believe that the situation is firmly within your control or are you starting to think that the situation is “getting out of control?” And if three more people start throwing trash cans at each other? Someone who is angry and hostile is more likely to have stronger external locus of control in the moment, not reduced.
  4. If this were true, this would be a good thing! Fewer depressive symptoms suggests an improvement in mental health. However, the question stem is alluding to the health disparities mentioned in the last sentence of the passage, and so the correct answer choice should be a negative outcome, not a positive one.

21)

A. The hippocampus is a part of the limbic system and is involved in the formation of memories, not the hormonal response to stress.

Which of the following findings would most weaken the authors explanation of the observed effects of class size reductions?

B. The medulla oblongata is the part of the brainstem responsible for respiratory muscle control as well as associated reflexes (sneezing, coughing, swallowing and vomiting). It does not initiate or begin the hormonal response to stress.

C. This is correct; the hypothalamus is the first component of the hypothalamic-pituitary-adrenal (HPA) axis responsible for the hormonal response to stress. The hypothalamus secretes corticotropin releasing hormone (CRH) which signals to the pituitary to secrete adrenocorticotropic hormone (ACTH) which then stimulates the release of cortisol (also known as the “stress hormone”) from the adrenal cortex.

D. The pons, like the medulla oblongata, plays a role in ventilation but does not initiate the endocrine response to stress. Specifically, the pons sets the ventilatory rate via control of the tidal volume. For more information on nervous system control of the respiratory system, check out https://jackwestin.com/resources/mcat-content/respiratory-system/regulation-by-nervous-control

Exam 3 Psychology/Sociology Section Passage 5

22)

  1. Per the passage, interpersonal problems were measured by trouble making friends and expressing their feelings. Like some of the other answer choices, these problems may be seen in individuals with depression, but they are not the primary evidence of depression; the answer choice “most directly” related to depression will have something to do with low mood.
  2. According to the third paragraph, externalizing problems were measured in part by how frequently the child got into arguments and how impulsive they were. A propensity to lash out or to act impulsively may be associated with depression in some individuals but is not direct evidence of depression. We’re looking for an answer suggesting low mood.
  3. Learning problems were marked by attention difficulties and task persistence. Trouble with staying focused and on task is seen with a number of disorders and is not the answer choice most directly related to depression.
  4. Internalizing problems were associated with anxiety, loneliness and sadness. These are more classically and directly associated with depression than any of the other answer choices, making it the correct answer for a “most directly” question.

23) Recall that a self-fulling prophecy involves an initial expectation influencing the outcome to match the expectation. Something about the expectation changes the situation in a way that makes the expected outcome more likely.

  1. This answer choice is correct; the expectation is the low expectations and the outcome is a behavior that reinforces the initial expectation. This suggests that the teacher will continue to have low standards which will continue to influence the students’ behavior such that they behave in a way that maintains the teacher’s low standards. This is the cycle of a self-fulfilling prophecy.
  2. In this answer choice the students are deeming the expectations unfair, but there is no mention or indication of the perceived unfair expectations influencing the outcome to match the high expectations. Without knowing if the students meet and reinforce the teacher’s high expectations, we cannot know if this is a self-fulfilling prophecy. If anything, students that believe the high expectations to be unfair are less likely to meet those expectations, not more likely.
  3. This answer choice is incorrect because the outcome (overcoming low standards) is different from the initial expectation (low expectations). Self-fulfilling prophecies would reinforce the expectation, not produce the opposite result.
  4. This answer choice is very similar to answer choice B and is similarly incorrect. There is no indication that the students are meeting the standards or reinforcing the high standards and, if anything, individuals who are frustrated with high standards are less likely to meet the standards, not more likely.

24) We have already seen the fundamental attribution error in this section; it describes the idea that we as individuals are more likely to attribute someone else’s behavior and actions to who they are as a person, and less likely to consider environmental or external factors influencing their actions.

  1. Social learning would involve other students and takes into account external factors making this answer choice incorrect.
  2. While cognitive dissonance would be an internal factor, emotional maturity is also an internal and not external factor. Whatever the teacher is overlooking should be a situational/environmental/external factor.
  3. This is an odd answer choice. Psychological disorders walk the line of what we might consider an internal factor in that some folks might say that the disorder is specific to the individual. However, we don’t typically consider psychological disorders a part of a person’s personality (barring the influence of personality disorders) or something that is clearly within someone’s control. In fact, we know that most psychological disorders are outside of the person’s control despite being modifiable with appropriate support. For example, if someone has post-traumatic stress disorder, we do not assume that they can decide what they find triggering or that their diagnosis is who they are as a person. There has to be a better answer choice.
  4. This is the best answer; it is most clearly a reliance on internal factors representing personality and character, and the overlooking of external and environmental factors. A dysfunctional personality is exactly that, part of a person’s personality and situational variables are synonymous with environmental factors.

25) Validity refers to the ability to approximate the true value of a measure and the ability to draw conclusions from the data obtained. In order to REDUCE CONCERN about validity, researchers should be able to replicate results and values obtained by different related measures.

  1. This answer choice describes the relationship between an independent and a dependent variable. Finding a correlation between the two does not inherently assuage concerns about the methods; the best way to reduce concern about the methods and whether conclusions can be drawn from the data obtained is to find a way to corroborate the measure or methods. Let’s keep looking at the other answer choices.
  2. Like the answer choice above, this answer choice addresses the relationship between the independent and dependent variable but does not actually provide support that the methods of the study, or the measure of the independent variable, are valid. The correct answer should show that the teachers’ ratings are capturing and approximating the true emotional problems. Also, this answer choice would actually cause more concern about the teacher’s ratings because more emotional problems correlating with less disruptions (negative correlation) would be the opposite of what is expected.
  3. This is the correct answer; it provides support that the teachers’ ratings are approximating the true value. Multiple measures of the independent variable are producing comparable and consistent results meaning the original measure is likely valid.
  4. This answer choice most resembles answer choice B; it is the opposite of what is expected and would make researchers question the validity of the measure more, not less.

Exam 3 Psychology/Sociology Section Discretes

26)

  1. This is the best definition of a dichotic listening task: two auditory stimuli, or messages, are presented to each ear and participants are asked to repeat what they heard. It is a measure of selective attention and was designed by Donald Broadbent in the 1950s.
  2. This is close but not quite right; the different stimuli are presented to each ear, not to one ear at the same time.
  3. By definition, subthreshold sounds would not be perceived. More importantly, there is no indication of multiple stimuli and nothing to suggest the study of selective attention.
  4. This answer choice has the participants dichotomizing the sound, but the key to the dichotic listening task is that the two auditory stimuli are already distinct and presented to each ear to test selective attention. Answer choice A best describes this and is the correct answer.

27) This question stem suggests that the teacher used operant conditioning and was using positive reinforcement to encourage students to complete problems.

  1. The loss of a behavior associated with operant conditioning when the reinforcement is removed is known as extinction; this is the best answer.
  2. Shaping describes the process of reinforcing behaviors that are similar to and that approximate the behavior of interest to guide the subject of the conditioning to the correct behavior. For example, if I want a child to clean their room, I might reward them when they start putting their toys in a pile, then again when they push the pile towards their toy box, and continue rewarding them as they place the toys into the toy box or wherever the toys belong.
  3. Stimulus discrimination is the ability to differentiate and respond differently to similar but distinct stimuli. The best example of this is your alarm clock versus a fire alarm; one is an annoyance in the morning and might cause you to reach over and hit the snooze button, while the latter is an indication of danger and often causes us to cover our ears and rush out of a building. They sound similar but we recognize them as distinct and respond accordingly.
  4. Stimulus generalization is responding the same way to similar but distinct stimuli. If you hand a small child a lemon and a lime to taste, they will likely make the same face when tasting each similarly sour citrus.

28) The question stem is focused on a longitudinal perspective that takes into account different life stages. Let’s see what the answer choices say.

  1. The biopsychosocial model takes into account biological, psychological and social factors that affect health behaviors but does not inherently account for longitudinal changes and different life stages, making this answer choice incorrect.
  2. Macrosociology is the large-scale counterpart to microsociology. While microsociology is focused on individual, interpersonal interactions, macrosociology is focused on overarching social structures including institutions, larger systems and entire societies. While the public health campaigns mentioned in the question stem are presumably targeting as many at-risk individuals as possible, the underlying idea is that an individual’s actions during adolescence/earlier life stage affects their health outcomes as an adult/later life stage. A macrosociological perspective is not focused on the longitudinal impact of an individual’s actions on their own future health.
  3. A social construction model would suggest that the individual’s risky behaviors are the result of their social interactions in a given environment or situation. While this may be contributing to risky behaviors, and in fact we know it does, this answer choice does not address the specific question; this would not explain the changing life stages and corresponding actions of the individual.
  4. The life course perspective is an approach in social epidemiology that takes into account the life experiences of an individual at varying life stages and considers how those experiences and decisions will impact future outcomes. This is precisely what is mentioned in the question stem making it the correct answer; risky behaviors during adolescence are associated with disorders in adulthood.

29) In order to challenge the theory that race and ethnicity are social constructs, the answer choice and hypothetical finding would have to present a biological or non-social explanation or data.

  1. This answer choice would support the idea that race and ethnicity are social constructs, not challenge it. Here they are shown to be inconsistently applied and dependent on who is applying the label suggesting a social source and definition.
  2. This is the correct answer because it would challenge a socially constructed definition of race and ethnicity. In this answer choice, there is a genetic and therefore biological difference between members of different racial and ethnic groups. This would challenge a strictly social definition of race and ethnicity, and if true, would suggest a biological contribution.
  3. Like answer choice A, this answer choice presents an inconsistent application of labels regarding race and ethnicity; this would correspond to a person-dependent label and would support a social construction of race and ethnicity, not call it into question.
  4. This answer choice is tempting because it has a seemingly non-social component, health status. However, the key difference between this answer choice and answer choice B is that answer choice B is strictly biological because it uses the genome as evidence, while this answer choice leaves a lot of variables unaccounted for which is precisely what we are learning has happened in the long history of medicine. This answer choice does not account for systematic differences in access to healthcare, potential systematic differences in environmental and occupational hazards across races and ethnicities, and much more. So while significantly different health status between two or more racial and ethnic groups may suggest that non-social and biological factors may be at play, it does not account for the numerous social and institutional factors that would also influence health status.

Exam 3 Psychology/Sociology Section Passage 6

30)

  1. Socialization is the lifelong learning of values, knowledge, customs, and attitudes held by a society, while out-groups describe a social group with which an individual does not identify. There is no indication in the last paragraph that the stressors experienced by racial and ethnic minorities is the result of learning about other racial and ethnic groups. For more on socialization, take a peek at the corresponding content page: https://jackwestin.com/resources/mcat-content/socialization/agents-of-socialization
  2. The last paragraph describes a hypothesis that prejudice and discrimination lead to increased alcohol consumption by racial and ethnic minorities. The underlying assumption or pathway is as follows: individual belongs to racial/ethnic minority -> their identity increases their exposure to prejudice and discrimination ->the increased exposure increases alcohol consumption. Some of their exposure to stressors is the result of the identity making answer choice B the best answer.
  3. This answer choice is the opposite of the relationship described in the final paragraph. The passage suggested that members with racial and ethnic minority identities would have differing outcomes as a result of their experiences, but this answer choice is describing the reverse by stating that health is affecting identity.
  4. This is incorrect; while the expectation in the last paragraph would align with lower socioeconomic status (serving as a proxy for the psychosocial stressors mentioned in the passage) and minority group status negatively affecting health outcomes, the findings do not support this expectation. The last sentence states that African Americans and Hispanic Americans have lower rates of alcohol use than their white counterparts, not higher rates. Therefore, the SES gradient and minority group status, both potentially contributing psychosocial stressors, do not appear to have an impact on the rates of alcohol consumption based on the data presented in the final paragraph.

31) According to the animal model described at the beginning of the passage, alcohol-dependent rats self-administered more alcohol than the nondependent rats following exposure to vapors. It also states that this response could be blocked by R121919.

  1. This answer choice is incorrect. While it correctly notes that the nondependent rats will not change their alcohol consumption following treatment, it reverses the effect on dependent rats. If ethanol dependent rats typically increase their alcohol consumption following vapor exposure, administration of a drug that blocks this response should produce a decrease in self-administration of alcohol; this answer choice shows an increase in alcohol consumption.
  2. Like answer choice A, this answer choice incorrectly shows alcohol consumption (shown by number of lever presses) increasing following vapor exposure and drug administration when it should be decreasing. Additionally, it shows the nondependent rats responding to the drug; they should not respond because they did not initially increase their alcohol consumption following vapor exposure.
  3. This answer choice is incorrect because it shows no response to the drug R121919 which should block the CRF1 receptor-mediated response to vapor exposure in ethanol-dependent rats. The last sentence of the second paragraph explicitly notes that this response should be blocked meaning no effect in rats with the response (this answer choice) is incorrect.
  4. This is the best answer because the nondependent rats do not respond to the drug since they do not initially respond with ethanol consumption to the vapor exposure, and the dependent rats do show a decrease in ethanol administration when given R121919. While the dependent rats normally increase alcohol consumption following vapor exposure due to a CRF1 receptor-mediated response, the administration of the drug R121919 which blocks this response decreases alcohol consumption.

32)

  1. A PET scan uses differential consumption of radioactive glucose and its metabolism to identify regions of increased activity. PET scans are often mentioned in the context of identifying tumors and cancers which tend to be much more metabolically active than their neighbors—can someone say hungry and rapidly dividing? They can also be used to determine which parts of the brain are activated during specific tasks. The question stem referred us to neural activity as mentioned in paragraph 3 which would be neural responses as they relate to rewards. The mesolimbic pathway is the “reward pathway” and would light up during a PET scan when paired with an activity that activates the reward pathway, meaning a PET scan would be a great imaging technique. This is the correct answer.
  2. Some students will find themselves tempted by this answer because EEGs measure electrical activity of the brain. However, EEGs do not provide data for specific regions and would not allow us to form an image of the brain associated with regions of activations. While EEGs are useful for diagnosing conditions such as seizure disorders, sleep disorders and brain injury, they are not useful for evaluating the activation of specific regions of the brain.
  3. MRIs provide information about the structure of the imaged region, but not the activity or in the case of the brain, activation of specific regions. An MRI is not to be confused with an fMRI or functional MRI. Unlike a regular MRI, a functional MRI can be used to measure activity because fMRIs use the differential flow of oxygenated blood to determine areas of increased brain activation. But alas, this answer choice is an MRI and not fMRI. Moving on!
  4. Like MRIs, CTs provide structural information about the region being imaged. You’ll learn how to read CTs in medical school, but even then, they won’t be able to determine activation.

33)

  1. The progression of experimentation to alcohol dependence due to conditioning will either involve the pairing of an unconditioned stimulus with a neutral stimulus to produce a conditioned stimulus (classical conditioning) or the use of reinforcements and punishments to increase the frequency of the behavior (operant conditioning). Stimulus generalization and stimulus discirimination are used to describe how a person or animal responds to stimuli other than the conditioned stimulus when the two are similar. An example we used in a previous question is responding differently to a fire alarm and a cell phone alarm, and responding similarly to the sour taste of a lime and a lemon to demonstrate stimulus discrimination and stimulus generalization respectively. They do not explain the progression of experimentation to alcohol dependence.
  2. Modeling involves learning via observation and is a type of observational learning. Classical and operant conditioning are types of associative learning that directly condition the subject to behave a certain way. You might sometimes hear modeling described as “vicarious conditioning” if the model that they are observing is learning via operant or classical conditioning, but it does not involve the direct conditioning of the person doing the observation. Additionally, shaping is used to help the subject begin to approximate the behavior of interest until they successfully complete the behavior of interest. This will occur when the behavior is not yet established and the person is rewarded for looking like they’re drinking or for being in an environment with alcohol, and unlikely to be a direct driver of alcohol dependence because the behavior (drinking alcohol) is already occurring.
    A flow chart for clarification: shaping + approximation of behavior (adolescent is rewarded for behaviors that approximate or make drinking more likely) -> subject finally performs the behavior (adolescent starts drinking alcohol) -> a reinforcement schedule is used to increase the frequency of the behavior (positive and negative reinforcement cement the behavior and may lead to alcohol dependence)
  3. This is the correct answer; both components of the answer increase how much the person drinks because, by definition, reinforcement increases the frequency of the behavior. Positive reinforcement is the addition of something that the subject likes to make them perform the behavior more often. The passage notes that adolescents may associate alcohol experimentation with the positive effects of alcohol on mood which would serve as the positive reinforcement. Negative reinforcement is the removal of something that the subject does not like that makes them more likely to perform the behavior. The alleviation or removal of negative emotions when drinking as noted in the last sentence of the third paragraph would be an example of negative reinforcement.
  4. An unconditioned response suggests that no learning has occurred yet and that drinking alcohol at a young age is an instinctual response. However, we know that adolescents do not randomly and instinctively start drinking alcohol; some learning takes place when the behavior is initiated.

34)

  1. Social facilitation describes the improvement on a task someone already finds easy or simple when performing that task in front of others. Drinking alcohol is not a task to be performed meaning this answer choice is incorrect and does not apply to the discussion in the passage.
  2. Obedience suggests that an authority figure is the one telling the adolescent to drink alcohol, but the passage notes that adolescents are drinking alcohol in their peer groups. Obedience and authority figures are not typically applied to peers so there must be a better answer.
  3. Peers would be an in-group, not an out-group. While drinking alcohol at a young age may be a type of deviance, this is not discussed in the passage and the question stem asks us to choose an answer based on the passage’s discussion of adolescent alcohol consumption.
  4. This is the best answer because conformity is the act of changing a behavior (or belief) to fit in with a group or community. Some students get really frustrated with this question because the passage mentions peers and this answer choice, the correct answer choice, classifies this group as a primary group. Recall that primary groups are formed when members have long, enduring, emotional ties with one another. The classic examples are family and close friends, but in this case can also include peers because they are seemingly interested in maintaining their bond. Also, secondary groups are usually brought together for some other reason (coworkers are a common example) that share some impersonal goal and are more apt to be temporary. As noted above, the peers in this passage are experimenting with alcohol together, suggesting a somewhat more intimate bond and making the label of primary group appropriate. For more on social groups, visit: https://jackwestin.com/resources/mcat-content/elements-of-social-interaction/groups

35)

  1. Additional risk factors would make it seem like prejudice and discrimination have a stronger impact on alcohol consumption, not a weaker one.
  2. The passage does not mention whether or not the models accounted for prejudice and discrimination separately or if they lumped the two together so we cannot comment on this. Additionally, one having a greater effect than the other does not explain why no relationship was found.
  3. Unmeasured protective factors would balance the risk of alcohol consumption with exposure to discrimination and prejudice, and could make it appear as though there is no effect when in reality there is an effect. This is the correct answer. Let’s walk through an analogy to make sure everyone is on the same page: we can all agree that children are more likely to make a mess when eating compared to adults because they have poorer coordination. If you put an adult and a child at the same table and measure the mess they each make when eating, but you don’t mention that another adult is helping feed the child, you will underestimate the mess that the child would have made without the assistance or protective factor. Unmeasured protective factors will minimize the measured effect of risk factors on outcomes.
  4. Like answer choice B, this answer choice is incorrect because it is not addressed or alluded to in the passage and also does not explain why no relationship was observed at all.

Exam 3 Psychology/Sociology Section Passage 7

36)

  1. The dependent variable is the outcome variable measured following exposure to the independent or manipulated variable. All households received the letter/flyer in the main and so it is something that was provided TO them, not the outcome.
  2. This is the correct answer. The outcome that the researchers measured was whether or not the households agreed to join the program. This outcome is measured across both the control group and the intervention group.
  3. All participating households received the phone call about the flyer and were asked to participate in the energy program. This is the way that the dependent variable data were collected, not the dependent variable itself.
  4. This is the independent variable, or the treatment. It is what was changed across the control and treatment arms, and is what is being compared to the dependent variable, not the dependent variable itself.

37)

  1. Bystander effect describes the phenomenon when someone is less likely to help someone in need because they expect someone else in the group, or another bystander, to help the person. No one is in need of assistance in this question stem.
  2. Social loafing occurs when people who are a part of a group do less or put in less effort because they will be graded or evaluated as a group. Many students reading this will identify with the following representative example: your teacher assigns you to a group project and someone in the group doesn’t do their share leaving others to “pick up the slack” so that they don’t get a poor grade on the group project. No social loafing in this question stem!
  3. This is the best answer because groupthink describes the tendency for groups to make decisions without considering alternatives and without as much critical reasoning in order to maintain harmony within said group. The example in the answer choice aligns with groupthink; if no one wanted to stir up the waters by disagreeing, then the four remaining folks simply agreed with the majority to keep the peace. Note that groupthink =/= group polarization! As noted above, groupthink involves reaching a conclusion as a group without much critical analysis, but group polarization involves reaching a consensus that is more extreme than the individual views of the members. For more on groupthink and group polarization, “think” about looking at their respective content pages: https://jackwestin.com/resources/mcat-content/elements-of-social-interaction/groups and https://jackwestin.com/resources/mcat-content/group-decision-making-processes/group-polarization
  4. Assimilation describes the process of someone from a minority or immigrant group learning and adopting elements of a new, majority and mainstream culture. The example in the question stem describes a single event and action; it is much too specific and lacks the element of cultural socialization found in assimilation.

38)

  1. Take a look at that! We just covered group polarization in our explanation for the last question (believe it or not, that was 100% unintentional…). Recall that group polarization involves a group reaching a more extreme consensus during discussion, and is more extreme than the views of its individual members at the start. This is exactly what happened in the study described in the question stem: members of the group held an idea, and after discussion, were even more convinced and even more sure of that idea. The fact that they are more sure than at the start indicates a stronger and more extreme opinion.
  2. Social facilitation occurs when someone does better at performing simple tasks in front of an audience. There are no tasks performed in front of an audience in the question stem.
  3. Peer pressure refers to when an individual feels pressured, often indirectly, to change their behavior to match the behavior of other group members. The group members described in the question stem are not changing their attitudes, they are becoming more sure of them. Answer choice A remains the best answer.
  4. Deindividuation occurs in groups and situations when an individual feels a loss of responsibility and identity which leads them to behave as a member of the group in ways that they would not behave if only acting as an individual. For example, someone might act more impulsively and aggressively during large events when they believe they are unlikely to be identified. This is not occurring in the study in the question stem; the best answer choice is A.

39)

  1. Classical conditioning involves the pairing of a neutral stimulus with an unconditioned stimulus that produces an unconditioned response. The neutral stimulus then becomes a conditioned stimulus that produces a conditioned response. In the description of their childhood, the participant noted observing her parents turn off the lights, but did not indicate that she was given any stimulus to turn off the lights. She was not classically conditioned.
  2. Operant conditioning involves the use of positive and negative reinforcements and punishments to determine the frequency of a desired or undesired behavior. There is no mention of her parents rewarding or punishing her for turning off the lights. In fact, she never mentions turning off the lights when leaving a room as a child. Operant conditioning is not at play.
  3. Operant conditioning and classical conditioning are types of associative learning. We’ve established that neither of the two apply to this particular example so associative learning is off the table. Let’s take a look at answer choice D.
  4. This is the correct answer because the individual in the question stem learned to do this as an adult because she watched, or observed, her parents perform and model the energy-saving behaviors.

Exam 3 Psychology/Sociology Section Passage 8

40)

  1. This is supported by the data in Table 1 and because this is a CANNOT question, it is incorrect. Starting from the bottom with 1973 and moving up to 2013, the percentage of respondents with “a great deal” of confidence in organized religion decreased from 43% to 25% (overall less confidence) while the percentage who had “very little” confidence increased from 7% to 17% (also a decrease in overall confidence in organized religion).
  2. This answer choice is also supported and therefore incorrect for this CANNOT question. In 1973, the percentage of people with “a great deal” and “quite a lot” of confidence in organized religion was 43% and 22% respectively for a total of 65% which is indeed greater than 60% as mentioned in the answer choice.
  3. Statistical significance is not provided in the table or the “note” associated with the table, so no conclusions about statistical significance can be drawn and we cannot say that the changes are statistically significant. Because this answer choice is not supported, it is the correct answer for this CANNOT question.
  4. Like answer choices A and B, this is supported by Table 1 and is therefore incorrect. The percentage of respondents with “very little” confidence in organized religion increased from 7% in 1973 to 17% in 2013 representing an overall increase.

41) Be careful here! The last question asked us to identify a statement that was not supported by one of the tables, while this question asks us to identify a statement that is supported by the other table. Keep the tables and “is” versus “is not” clear when answering these questions.

  1. This is the correct answer; this statement only evaluates the data from one age group to the data from another age group and does not extrapolate beyond what is presented in the table. A smaller percentage of younger adults (easily estimated by 18-29 year of age, 33%) attend weekly religious services compared to a higher percentage of older adults (easily estimated by the 65+ age group, 53%). Some of you might be asking how we decided which age range would constitute younger and older adults. The data in the table make it easier because the two groups in the middle age range have very similar percentages and would roughly cancel one another out if considered. Also, we just called them middle aged so we wouldn’t worry about them for this question.
  2. No other factors are mentioned with respect to the data shown in Table 2, and as such no conclusion can be drawn about the relative importance of any other factors that might be influencing attendance of religious services.
  3. This answer choice is incorrect for two reasons. First, if we assume that the data obtained are only from Americans who do attend religious services, then the data do not take into account all of the Americans who do not attend religious services at all and we cannot say what percentage of all Americans do or do not attend weekly religious services. This interpretation of the sampling data is supported by the statement in the passage that the survey was performed AT services. Second, even if we assume that the data are from individuals who do and do not attend services, there is still no indication as to the relative sizes of the different age groups and we would not be able to estimate what percentage of the total American population attends weekly services without more information. Both explanations are provided to ensure that you can walk through similar questions with ease, but the good news is that you really only need one reason to decide that an answer is incorrect on test day.
  4. Like the discussion from the previous question, without a note or indication of statistical significance associated with the table, no conclusions about the statistical significance of the data can be drawn. This answer choice is incorrect and answer choice A is the correct answer.

42) A conflict theoretical analysis is another way of saying that it aligns with conflict theory. Conflict theory focuses on the unequal distribution of resources and power differentials across society, the struggle for power, and the creation and maintenance of social order. This theory is rooted in the works of Karl Marx. If you read “conflict theory” on test day, think “class conflict.” If you’re still, ahem, conflicted about how to approach conflict theory, take advantage of the resource that follows: https://jackwestin.com/resources/mcat-content/theoretical-approaches/conflict-theory

  1. This statement would be consistent with a conflict theory lens because conflict theory considers the importance of social control and if religion were an instrument of that social control, then it could be used in the maintenance of social order.
  2. Like the answer choice above, this statement would also be consistent with a conflict theory approach because if true, it would propagate the unequal distribution of resources and address the power differential described above.
  3. This statement is also consistent with conflict theory because passive acceptance of material resources would lead to the maintenance of the status quo and of unequal distribution of said resources.
  4. This is the only answer choice that would contradict a conflict theory approach to the results provided in the passage. If religion increased social solidarity, it would decrease the conflict between classes and remember, if you read “conflict theory” think “class conflict.” This is the correct answer.

43)

  1. This is incorrect; the use of a website for a religious organization or congregation does not constitute the secularization of religion.
  2. This is correct; the religious leader recognized that there was a difference in the attendance of religious services by members of different cohorts (younger vs older adults). The religious leader recognized these generational differences and tried to address the difference by using a means of communication that followed a similar generational pattern. Younger people were not attending religious services as often, but they likely use the internet more and as such can be preferentially attracted by the use of a website for the congregation.
  3. Some folks might be tempted because of the period effects mentioned in this answer choice. Period effects apply to everyone living at a given point in time and so without knowing whether the answer choice is referring to current period effects or period effects from a time when younger people were not yet born, this answer choice is incorrect. Additionally, this answer choice tries to tie in “everyday life” which is not the focus of this question. The correct answer is B because it correctly identifies a difference between cohorts without extrapolating too much.
  4. No value judgment is made on the importance of either religion or technology in the question stem.

Exam 3 Psychology/Sociology Section Discretes

44)

  1. If hyperexcitation is causing neuronal death, then the magnitude of a given action potential cannot be what’s responsible for the differential damage to the hippocampus in comparison to the cortex. Recall that once the potential difference in a neuron crosses the threshold, the action potential will propagate throughout the neuron. Either there is or there is not an action potential so difference in magnitude does not apply.
  2. NMDA receptors, or N-methyl-D-aspartate receptors, are a type of glutamate receptor. If there are more glutamate receptors on neurons of the hippocampus than there are on neurons within the cortex, then the neurons in the hippocampus are more prone to glutamate-mediated neuronal death via hyperexcitation.
  3. The question stem specifies a hyperexcitation of the neurons which would correspond to a more positive postsynaptic potential, not a more negative potential.
  4. We have no reason to believe that neurotransmitters diffuse more slowly across the synaptic cleft in the hippocampus than in the cortex.

45)

  1. INTERgenerational mobility refers to a change in social class across different generations, for example a parent and their child belonging to different social classes. The question stem refers to a single individual changing social classes.
  2. INTRAgenerational mobility would correspond to a single individual changing social classes during their lifetime which is exactly what is described in the question stem. This is the correct answer.
  3. Structural mobility would represent a structural or collective change that results in the change of an entire group’s social class. The question stem specifies a single individual changing social classes, not a group and not accompanied by wider societal changes. This answer choice is incorrect.
  4. Horizontal mobility describes a change in position without a change in social class. An example of this would be changing the place of employment. The individual in the question stem did change social class so this answer is incorrect.

46)

  1. This answer choice is too general and incorrect. Swabbing of the same urine from the male Strain A mice produced different results in the female Strains B and C mice, so the male mice are not affecting all female mice in the same way.
  2. This answer choice could explain the results seen in the question stem making it correct. The two strains of female mice responded differently to the same urine swabbing from nearly genetically identical mice, so the difference had to be in the female mice strains. The Strain B mice had increased uterine weight when their nostrils were swabbed with the Strain A urine which would support a puberty-accelerating event (enlargement of a reproductive organ) secondary to pheromone or chemical signaling.
  3. Strain C mice had no change in uterine weight; it was the Strain B mice that had increased uterine weight making this answer incorrect.
  4. The question stem specifies that the mice are inbred which means that they are nearly genetically identical. This would virtually eliminate genetic variation as a potential confounder.

47)

  1. A self-fulfilling prophecy would correspond to a set of expectations making the expected outcome more likely such as thinking you won’t be able to learn the newest social media dance and then struggling to learn the dance. This is not occurring in the question stem.
  2. Self-verification is our tendency to look for information that aligns with our own sense of self, self-concept and self-identity. We are also more likely to believe the information that is consistent with our own view. In the question stem, the participants are rating how close and accurate their partners’ ratings of them were which would indeed be an example of self-verification.
  3. Self-serving bias refers to the tendency to attribute positive events in our lives to internal favors and negative events in our lives to external factors to “serve” our own positive image. This is not occurring in the question stem.
  4. Self-efficacy refers to someone’s perceived ability to complete a task. No one in the study rated whether they thought they had the skills and ability to do something so this answer choice does not apply, and answer choice B remains the best answer.

Exam 3 Psychology/Sociology Section Passage 9

48) The two personality traits mentioned in Study 2 are conscientiousness and neuroticism. For more on the theories or personality, take a look at the associated content page and browse the table below. Now onto the answer choices!

Content page: https://jackwestin.com/resources/mcat-content/personality/theories-of-personality

Which of the following findings would most weaken the authors explanation of the observed effects of class size reductions?

  1. The Five Factor Model of personality traits, or the Big Five model includes openness, conscientiousness, extroversion, agreeableness and neuroticism. You can use the “OCEAN” mnemonic to help you remember the five traits. Both conscientiousness and neuroticism are included above making this the best answer choice.
  2. Freud’s psychodynamic or psychoanalytic approach revolves around the conscious and unconscious mind, namely the differences and interplay between the id, the ego and the superego.
  3. This was once such a popular quiz to take in school (I know, I know, a quiz, popular?) and as such many of you may be familiar with the inventory and even know your own type. For those who are not familiar, the Myers-Briggs inventory is a type theory that considers how individuals interact with the world around them with respect to intro/extroversion, degree of attention to detail versus looking at the larger picture, whether they are more logic or emotion oriented, and their degree of planning ahead. We don’t see these characteristics mentioned in the context of Study 2 so this answer choice is incorrect.
  4. The biopsychosocial model takes into account, as the name implies, biological, psychological and social factors affecting an individual’s health, development and so much more. It does not limit itself to specific personality traits and because it does not specifically address conscientiousness and neuroticism, it is incorrect.

49) The question stem is sending us back to Study 1 where the author notes that aggression and oppositional behaviors were associated with hunger. The correct answer and neurotransmitter will have a known relationship with aggression and hunger.

  1. While acetylcholine is associated with movement and muscle contraction, that movement is not inherently aggressive.
  2. GABA is an inhibitory neurotransmitter and is not associated with aggression or hunger.
  3. Endorphins are endogenous pain killers. They are not directly associated with aggression or hunger, despite the fact that many folks might get cranky if in pain.
  4. Serotonin is the only neurotransmitter that directly regulates aggression and hunger making it the best answer. Serotonin regulates mood (including aggression), appetite/hunger and bowel movements, sleep, blood clotting and even bone health! It’s the neurotransmitter equivalent of a jack of all trades.

50) The two personality traits mentioned in Study 2 are conscientiousness and neuroticism. Conscientiousness refers to planning and responsibility while neuroticism relates to anxiety and insecurity.

  1. If conscientiousness is related to planning and responsibility, then food insecurity would be negatively associated or correlated with conscientiousness. No need to read the rest of the answer choice on test day.
  2. Like answer choice A, this answer choice is incorrect because conscientiousness is considered a positive trait related to responsibility and planning and would be negatively associated with food insecurity.
  3. The first half of this answer choice is correct, but the second half is incorrect. Neuroticism refers to anxiety and insecurity in the face of stress. This would be positively correlated with the high stress of food insecurity.
  4. Food insecurity would be associated with less planning (conscientiousness) and higher anxiety and insecurity (neuroticism). This answer choice correctly captures this negative association between food insecurity and conscientiousness and positive association between food insecurity and neuroticism.

51) Referring back to the paragraph on Study 1, we see that the researchers believe that disruption of homeostasis is the underlying motivation.

  1. According to the incentive theory, underlying motivation for behaviors is primarily extrinsic and focused on being rewarded and avoiding punishment. There is no homeostasis to be maintained in the incentive theory.
  2. This is the correct answer because drive reduction theory is centered on the need to maintain physiological homeostasis. Drives help achieve biological homeostasis—they “drive” you to meet biological needs such as drinking when thirsty—while motives are the result of social and psychological mechanisms. Homeostasis is the key word in both the passage and the theory of interest.
  3. According to the expectancy-value theory, the degree of motivation is related to the degree of expected success and the value of that success. Maintenance of homeostasis is not the objective of the expectancy-value theory.
  4. The self-determination theory focuses on autonomy, competence and relatedness, not homeostasis. Only answer choice B directly addresses the hypothesized motivation, homeostasis.

52)

  1. Role engulfment occurs when one role or label engulfs or overtakes someone’s identity. A classic example of this is someone becoming sick and the “sick” label becomes the major component of their identity, restricting their own self-image. This is not the same as struggling as a parent to meet your children’s needs as well as your own.
  2. Role confusion is a part of one of Erikson’s stages of development, specifically the Identity vs Role Confusion stage during adolescence (~12-18 years of age) and is the result of not successfully forming one’s own identity after exploring various roles and ideas. This is not what is described as happening to the parents in the passage as they struggle to meet their own needs and the needs of their children.
  3. Role conflict refers to the difficulty one might experience when trying to manage multiple roles. For example, your friend wants you to go watch a movie with them because they are bored, your mom wants you to stay and do the dishes, and you have an exam tomorrow and need to study. You have three different roles all vying for your time and attention, and it’s difficult to keep up with all three. There are not multiple roles described in the passage; the single role is the parental role and the parent is struggling to meet their children’s needs as well as their own.
  4. Role strain refers to difficulty managing a single role and is indeed what is described in the passage, making it the correct answer. The parents in the passage are noted to be more likely to have depression and antisocial tendencies, and the researchers suggest that these parents are struggling to meet their children’s needs as well as their own needs. There is a single role here, the role of parent, and these parents are struggling to address the competing needs and responsibilities within that role: caring for themselves and caring for their children.

Exam 3 Psychology/Sociology Section Passage 10

53) An easy way to determine independent and dependent variables in a study is that the general format for describing the two is to note the effect OF the independent variable ON the dependent variable. Other words to look for are “explanation for” and what follows is usually the dependent variable, and “contributes to” followed by the dependent variable.

  1. This is the correct answer; the outcome that the researchers were interested in and measured was cardiac deaths. They were looking to identify the possible effects OF holidays (specifically Christmas and the New Year) and cold weather ON cardiac deaths.
  2. The researchers were interested in and evaluated the daily mortality and causes of death, not physician observations.
  3. As noted in the explanation for answer choice A, this was one of the independent variables, not the dependent variable.
  4. Similar to cold/winter weather, holiday practices and the potential effects of holidays was an independent variable, not the dependent variable.

54) The researchers suggest looking at the effect of inequitable access to healthcare on cardiac deaths so the correct answer should have a clear effect on access to healthcare facilities.

  1. While gender may influence healthcare-seeking behaviors, this effect is much smaller than that of the correct answer choice and there is no mention in the passage of disproportionate cardiac deaths across gender.
  2. Non-natural causes of death will not affect access to healthcare; once the person has passed away, very little if anything can be done.
  3. Like gender, age may affect access to healthcare but does not have the greatest impact of the answer choices provided and in fact, differences in access to healthcare across age groups is often mediated by what is the correct answer for this question, socioeconomic status.
  4. Socioeconomic status can and does significantly impact access to healthcare facilities. If people cannot afford to seek medical care or if they are unable to arrive at healthcare facilities, then there will be unequal access to healthcare; the researchers would likely include SES as a variable in future studies.

55) Symbolic interactionism studies the way individuals interact with one another through words, actions, symbols, etc. These symbols have shared meanings and are the result of social interactions.

  1. If it is a natural increase, it is biological and not the result of social interactions or symbolic meanings that are associated with actions. This answer choice is incorrect.
  2. Holiday practices and rituals would be social interactions with shared symbolism and meaning, supporting a symbolic interactionism approach to the observations in the passage. This is the best answer choice.
  3. Travel and corresponding increases in external environmental factors is more representative of functionalism than social interactionism. Functionalism states that different aspects and parts of society serve specific functions which may be manifest if desired and expected, or latent if unexpected/unintentional. This answer choice suggests that travel functions to increase external factors which fits nicely with functionalism making this answer choice incorrect.
  4. The mention of macro-structural factors indicates a macrosociological approach to the interpretation of the data. Macrosociology emphasizes the impact of overarching social structures such as institutions and entire societies, not the symbolic interactions between individuals.

56) Because this is a LEAST question and the researchers want to study the effect of emotions on cardiac mortality, the correct answer will be the one that does not have an emotional component.

  1. Learned helplessness describes feeling hopeless or without control after repeated negative outcomes. This hopelessness would absolutely have an emotional component and could theoretically affect cardiac mortality.
  2. A locus of control, be it internal or external, can affect someone’s emotions. For example, having a stronger external locus of control would make someone more prone to experiencing learned helplessness and affect mood via that route. The correct answer should not have an emotional component and this one does, so this answer choice is incorrect.
  3. The fundamental attribution error describes the increased likelihood of attributing another person’s behavior to a disposition or internal factors rather than external or situational factors. There is no inherent emotion in this cognitive error and as such this is the best answer and the one that is least likely to affect cardiac mortality via emotions.
  4. Self-efficacy is what we believe we are capable of and whether we believe ourselves to have the skills and abilities to complete a given task. If we have low self-efficacy, we might feel less motivated or “down;” self-efficacy can affect emotions making this answer choice incorrect. For more on self-concept, review https://jackwestin.com/resources/mcat-content/self-concept-and-self-identity-and-social-identity/the-role-of-self-esteem-self-efficacy-and-locus-of-control-in-self-concept-and-self-identity

Exam 3 Psychology/Sociology Section Discretes

57) Ah, the hidden curriculum, this is a term you will hear again in medical school. The hidden curriculum is the set of ideas, lessons and norms that are reinforced but not acknowledged as such by an academic institution. For example, the school will pair you with a mentoring physician or preceptor; if your preceptor rewards you for muddling through a cold while in clinic or on rotations, they are teaching you that work comes before your own health. This is an unintended or unacknowledged consequence of the school intentionally pairing you with a physician mentor.

  1. The manifest function of an educational institution is the explicitly acknowledged purpose or intended effect of deliberate actions taken by the institution. This is the opposite of a hidden curriculum where the lessons and their role are not acknowledged or expressly intended.
  2. The hidden curriculum is not an equalizing force or function in schools. The idea is that the students are learning something that is not explicitly intended. These lessons will not “level the playing field” and are about the vertical transmission of knowledge, behaviors and expectations while equalization is focused on achieving horizontal parity.
  3. This is correct. A latent function if not explicitly acknowledged, is unintended or unexpected. The hidden curriculum is not explicitly taught or enforced and is therefore unrecognized making it a latent function of education systems.
  4. The school is not discriminating or treating one group of students differently than another group of students; the hidden curriculum is, theoretically, taught to any students exposed to the institution where it is taught.

58) If people need to have their eyes open to balance on one foot, there must be some interaction between visual cues and balance.

  1. Have you ever paused to wonder why cars seem to whoosh past you once they get close to your car while driving? This phenomenon relates to motion parallax which is a monocular cue related to depth perception. Objects that are farther away moving at a given speed seem to move more slowly and objects that are closer appear to move faster when traveling at that same speed.
  2. Sensory interaction describes the idea that the different senses may influence one another which is exactly what is described and alluded to in the question stem; vision is influencing balance or the vestibular sense.
  3. The vestibular sense is our sense of balance. While it is absolutely at play when trying to balance on one foot, it does not address the need to have our eyes open in order to remain balanced. Answer choice B is a better answer because it explains the relationship between the two senses.
  4. Perceptual maladaptation would correspond to a harmful trait or response, but there is nothing in the question stem to indicate that needing to have your eyes open to balance on one foot is harmful or undesirable. You and I may want to be able to balance on one foot with our eyes open, but our desires are not enough to make it a correct answer.

59) For a review of the different Gestalt principles, head over to the corresponding content page: https://jackwestin.com/resources/mcat-content/perception/gestalt-principles

A. The law of similarity states that we will perceive objects or elements that resemble each other or are similar as being grouped. For example, when we see the grey circles and black circles in the figure below, our mind will group the circles into alternating lines. You might be saying “hey, but a C and an O sure do look similar to me!” In fact they do, but the question stem notes that the C is read as an O, so the result is not their visual grouping as would be the case if they were side-by-side, but the seeming conversion of one into the other.

Which of the following findings would most weaken the authors explanation of the observed effects of class size reductions?

B. The law of closure states that when we see incomplete objects, we perceive them as completed. When we compare the letters C and O, the C might look like an O if our minds perceive the C as incomplete and then fills in the remaining part to look like an O. Below is another example; our minds will fill in the lines of the IBM logo:

Which of the following findings would most weaken the authors explanation of the observed effects of class size reductions?

C. The proximity Gestalt principle states that if objects are close to one another, they will be perceived as part of a group.

Which of the following findings would most weaken the authors explanation of the observed effects of class size reductions?

D. The law of symmetry explains that when we see symmetrical objects, they are likely to be seen as a unified grouping.

Which of the following findings would most weaken the authors explanation of the observed effects of class size reductions?